Đến nội dung

Hình ảnh

Topic nhận đề Bất đẳng thức


  • Chủ đề bị khóa Chủ đề bị khóa
Chủ đề này có 15 trả lời

#1
E. Galois

E. Galois

    Chú lùn thứ 8

  • Quản lý Toán Phổ thông
  • 3861 Bài viết

Chuyển nhanh đến:
1) Điều lệ
2) Đăng kí thi đấu
3) Lịch thi đấu và tổng hợp kết quả

Topic này dùng để BTC nhận đề thi từ các toán thủ thi đấu.

Điều 3. Phương thức thi đấu, cách tính điểm:
a. Phương thức thi đấu:

- Trước mỗi trận, các toán thủ nộp đề cho BTC, BTC chọn 1 đề thi đấu. Đề thi được chọn là của toán thủ nào thì toán thủ đó gọi là toán thủ ra đề.Toán thủ ra đề không phải làm bài. (BTC đảm bảo nguyên tắc mỗi toán thủ chỉ được chọn đề nhiều nhất 1 lần). Toán thủ đã được chọn đề 1 lần thì những trận sau đó không cần phải nộp đề nữa.
- Trong trường hợp đến hết ngày thứ Tư hàng tuần mà không có toán thủ nào nộp đề, BTC sẽ chỉ định toán thủ có SBD nhỏ nhất (chưa có đề được chọn) phải ra đề.
...

b. Cách tính điểm
...

+ Nếu ra đề sai, đề không đúng chủ đề định sẵn, đề vượt quá cấp học hoặc không giải được đề mình ra, toán thủ ra đề được −30 điểm.
+ Nếu đến lượt mà không ra đề được −20 điểm.
+ Ra đề mà không post đáp án đúng thời gian được −10 điểm
...


Điều 6. Quy định đề bài:
a. Nội dung:
-
Mỗi bộ đề bao gồm 1 câu không copy nguyên văn từ đề thi Olympic quốc gia trở lên.
b. Hình thức:
- Đề bài được gõ Latex rõ ràng.



BTC yêu cầu các toán thủ nộp đề về Bất đẳng thức. Đề cần nộp cùng đáp án

Các toán thủ khi thi đấu, cứ yên tâm rằng, sau khi đánh máy là đề đã được lưu, BTC đã nhận được đề của bạn, có điều bạn không nhìn thấy được mà thôi. Bạn nên mừng vì điều này, như thế các toán thủ khác không thể biết trước đề của bạn được.

Bạn cũng nên sử dụng chức năng xem trước của diễn đàn để sửa các lỗi Latex trước khi gửi bài, vì gửi rồi sẽ không xem và sửa lại được nữa.

1) Xem cách đăng bài tại đây
2) Học gõ công thức toán tại: http://diendantoanho...oạn-thảo-latex/
3) Xin đừng đặt tiêu đề gây nhiễu: "Một bài hay", "... đây", "giúp tớ với", "cần gấp", ...
4) Ghé thăm tôi tại 
http://Chúlùnthứ8.vn

5) Xin đừng hỏi bài hay nhờ tôi giải toán. Tôi cực gà.


#2
tran thanh binh dv class

tran thanh binh dv class

    Trung sĩ

  • Thành viên
  • 138 Bài viết
Cho $a,b,c>0$. Chứng minh rằng:
$\frac{a(b+c)}{a^2+2bc}+\frac{b(c+a)}{b^2+2ca}+\frac{c(a+b)}{c^2+2ab}\leq 2+\frac{2(a-b)^2(b-c)^2(c-a)^2}{(a^2+2bc)(b^2+2ac)(c^2+2ab)}$

Hình đã gửi


#3
namcpnh

namcpnh

    Red Devil

  • Hiệp sỹ
  • 1153 Bài viết
Toán thủ namheo1996 xin ra đề:

ĐỀ:
Cho a,b,c là các số thực dương.Nếu $n\geq 1$ ,chứng minh rằng:

$\frac{a^{n+1}}{b+c}+\frac{b^{n+1}}{a+c}+\frac{c^{n+1}}{a+b}\geq (\frac{a^{n}}{b+c}+\frac{b^{n}}{a+c}+\frac{c^{n}}{a+b})\sqrt[n]{\frac{a^{n}+b^{n}+c^{n}}{3}}$.

Bài giải:

Bất đẳng thức cần chứng minh được viết lại thành:

$\sum (\frac{a^{n +1}}{b+c}+a^{n})-\sqrt[n]{\frac{a^{n}+b^{n}+c^{n}}{3}}\sum \frac{a^{n}}{b+c}\geq a^{n}+b^{n}+c^{n}$

<=>$(a+b+c-\sqrt[n]{\frac{a^{n}+b^{n}+c^{n}}{3}})\sum \frac{a^{n}}{b+c}\geq a^{n}+b^{n}+c^{n}$

Sử dụng bất đẳng thức Cauchy-Swcharz ta có:

$\frac{a^{n}}{b+c}+\frac{b^{n}}{a+c}+\frac{c^{n}}{a+b}\geq \frac{(a^{n}+b^{n}+c^{n})^{2}}{a^{n}(b+c)+b^{n}(a+c)+c^{n}(a+b)}$

Bài toán quy về chứng minh:

$a+b+c-\sqrt[n]{\frac{a^{n}+b^{n}+c^{n}}{3}}\geq \frac{a^{n}(b+c)+b^{n}(a+c)+c^{n}(a+b)}{a^{n}+b^{n}+c^{n}}$

<=>$\frac{a^{n+1}+b^{n+1}+c^{n+1}}{a^{n}+b^{n}+c^{n}}\geq \sqrt[n]{\frac{a^{n}+b^{n}+c^{n}}{3}}$

<=>$(\frac{a^{n+1}+b^{n+1}+c^{n+1}}{3})^{\frac{1}{n+1}}\geq (\frac{a^{n}+b^{n}+c^{n}}{3})^{\frac{1}{n}}$

Hiển nhiên bất đẳng thức trên đúng theo bất đẳng thức trung bình lũy thừa.

Dấu "=" xảy ra <=>a=b=c

Vậy bất đẳng thức đã được chứng minh.

Cùng chung sức làm chuyên đề hay cho diễn đàn tại :

Dãy số-giới hạn, Đa thức , Hình học , Phương trình hàm , PT-HPT-BPT , Số học.

Wolframalpha đây


#4
luuxuan9x

luuxuan9x

    Sát thủ có khuôn mặt trẻ thơ

  • Thành viên
  • 78 Bài viết
Mình ra đề trận 3:

Cho a,b,c là các số không âm thoả điều kiện không có bất cứ 2 số nào đồng thời bằng 0.Chứng minh rằng:

$\sqrt{\frac{a}{b+c}}+\sqrt{\frac{b}{a+c}}+\sqrt{\frac{c}{a+b}}+\frac{ab}{(a+b)^{2}}+\frac{bc}{(b+c)^{2}}+\frac{ac}{(a+c)^{2}}\geq \frac{9}{4}$

Bài giải:

Sử dụng bất đẳng thức Holder ,ta có:

$(\sum \sqrt{\frac{a}{b+c}})^{2}[\sum a^{2}(b+c)]\geq (\sum a)^{3}$

Từ đó suy ra:

$\sum \sqrt{\frac{a}{b+c}}\geq \sqrt{\frac{(\sum a)^{3}}{\sum a^{2}(b+c)}}\geq \sqrt{\frac{(\sum a)^{3}}{\sum a^{2}(b+c)+3abc}}=\frac{\sum a}{\sqrt{\sum ab}}$ (1)

Mặc khác theo bất đẳng thức Cauchy-Schwarz ta lại có:

$\sum \frac{ab}{(a+b)^{2}}\geq \frac{1}{2}\sum \frac{ab}{a^{2}+b^{2}}=\frac{1}{4}\sum \frac{(a+b)^{2}}{a^{2}+b^{2}}-\frac{3}{4}\geq \frac{(\sum a)^{2}}{2\sum a^{2}}-\frac{3}{4}$ (2)

Từ (1) và (2) suy ra:

$\sum \sqrt{\frac{a}{b+c}}+\sum \frac{ab}{(a+b)^{2}}\geq \frac{\sum a}{\sqrt{\sum ab}}+\frac{(\sum a)^{2}}{2\sum a^{2}}-\frac{3}{4}$

Và như vậy, bằng cách sử dụng kết quả này , ta đưa được bài toán về chứng minh:

$\frac{a+b+c}{\sqrt{ab+bc+ca}}+\frac{(a+b+c)^{2}}{2(a^{2}+b^{2}+c^{2})}\geq 3$ (*)

Gọi VT là vế trái của (*) khi đó ta có:

VT=$\frac{a+b+c}{2\sqrt{ab+bc+ca}}+\frac{a+b+c}{2\sqrt{ab+bc+ca}}+\frac{(a+b+c)^{2}}{2(a^{2}+b^{2}+c^{2})}$

$\geq 3\sqrt[3]{\frac{(a+b+c)^{4}}{8(ab+bc+ca)(a^{2}+b^{2}+c^{2})}}\geq 3\sqrt[3]{\frac{(a+b+c)^{4}}{(a^{2}+b^{2}+c^{2}+2ab+2ac+2bc)^{2}}}=3$

=> (*) luôn đúng

Dấu "=" xảy ra <=> a=b và c=0 hoặc các hoán vị.

Vậy bất đẳng thức đã cho được chứng minh.

#5
cool hunter

cool hunter

    Thiếu úy

  • Thành viên
  • 544 Bài viết
Đ:
Cho $a,b,c$ là các số thực dương thỏa mãn: $ab+a+b=b$. C/m: $\frac{3a}{b+1}+\frac{3b}{a+1}+\frac{ba}{b+a}\leq a^{2}+b^{2}+\frac{3}{2}$(*).
Đáp ân:
Đặt $t=a+b$ => $ab =3-t$ & $a^{2} +b^{2} =t^{2} -2(3-t) =t^{2} +2t -6$.
Vì $(a+b)^{2}\geq 4ab\Rightarrow t^{2}\geq 4(3-t)\Leftrightarrow t^{2}+4t-12\geq 0\Leftrightarrow t\geq 2(vì t>0)$.
Khi đó:
(*)$\Leftrightarrow \frac{3(a^{2}+b^{2})+3(a+b)}{(a+1)(b+1)}+\frac{ab}{a+b}-(a^{2}+b^{2})\leq \frac{3}{2}$
$\Leftrightarrow \frac{3t^{2}+6t-18+3t}{4}+\frac{3-t}{t}-t^{2}-2t+6\leq \frac{3}{2}$
$\Leftrightarrow -t^{2}+t+\frac{12}{t}\leq 4(1)$
Xét hàm số: $f(t)=-t^{2}+t+\frac{12}{t}$ với $t\geq 2$
Ta có: $f'(t)=-2t+1-\frac{12}{t^{2}}<0\forall t\geq 2$
$\Rightarrow f(t)\leq f(2)=4\forall t\geq 2$ =) (1) đúng => đpcm
Đẳng thức xảy ra khi $a=b=1$

Thà đừng yêu để giữ mình trong trắng

Lỡ yêu rôì nhất quyết phải thành công

                                                                 


#6
L Lawliet

L Lawliet

    Tiểu Linh

  • Thành viên
  • 1624 Bài viết
Bài toán: Cho $a$, $b$, $c$ là các số thực dương. Chứng minh rằng: $\dfrac{a^3}{b^2-bc+c^2}+\dfrac{b^3}{c^2-ca+a^2}+\dfrac{c^3}{a^2-ab+b^2}\geq \dfrac{3\left ( ab+bc+ca \right )}{a+b+c}$.

Lời giải:
Ta có các mẫu thức của $VT$ đều dương, chẳng hạn $b^2-bc+c^2=\left ( b-\dfrac{c}{2} \right )^2+\dfrac{3c^2}{4}>0$, hơn nữa $a$, $b$, $c$ đều dương nên ta đặt $A$ là biểu thức ở $VT$ và $S=\left ( a^2+b^2+c^2 \right )^2$, ta biến đổi như sau:

$\sqrt{S}=\sqrt{\dfrac{a^3}{b^2-bc+c^3}}\sqrt{\left ( b^2-bc+c^2 \right )a}+\sqrt{\dfrac{b^3}{c^2-ca+a^3}}\sqrt{\left ( c^2-ca+a^2 \right )b}+\sqrt{\dfrac{c^3}{a^2-ab+b^3}}\sqrt{\left ( a^2-ab+b^2 \right )c}$.

Từ biểu thức trên, áp dụng bất đẳng thức $Cauchy-Shwarz$, ta có:

$S\leq A\left ( ab^2+ac^2+bc^2+ba^2+ca^2+cb^2-3abc \right )\left ( 1 \right )\\ \Leftrightarrow A\geq \dfrac{S}{ab^2+ac^2+bc^2+ba^2+ca^2+cb^2-3abc}\left ( 2 \right )$
(Vì ta đã chia hai vế của $\left ( 1 \right )$ cho một biểu thức dương).

Ta sẽ chứng minh $VP$ của $\left ( 2 \right )$ không bé hơn $a+b+c$. Thật vậy, điều đó tương đương với:
$a^4+b^4+c^4+abc\left ( a+b+c \right )\geq ab^3+ac^3+bc^3+bc^3+ba^3+ca^3+cb^3\left ( 3 \right )$
Không mất tính tổng quát, ta giả sử $a\geq b\geq c>0$, ta có:
$a^2\left ( a-b \right )^2+\left ( a-b \right )^2\left ( a+b \right )\left ( b-c \right )+c^2\left ( c-a \right )\left ( c-b \right )\geq 0\\ \Leftrightarrow a^4+b^4+c^4+abc\left ( a+b+c \right )\geq ab^3+ac^3+bc^3+ba^3+ca^3+cb^3$
Như vậy $\left ( 3 \right )$ đúng.
Kết hợp $\left ( 2 \right )$ với $\left ( 3 \right )$, ta có $A\geq a+b+c$ và ta phải chứng minh: $a+b+c\geq \dfrac{3\left ( ab+bc+ca \right )}{a+b+c}\left ( 4 \right )$.
Thật vậy, ta đã biết $a^2+b^2+c^2\geq ab+bc+ca$ nên $\left ( a+b+c \right )^2\geq 3\left (ab+bc+ca \right )$, mà $a+b+c>0$ nên ta có $\left ( 4 \right )$, suy ra $Q.E.D$ $\blacksquare$

Thích ngủ.


#7
Joker9999

Joker9999

    Thiếu úy

  • Thành viên
  • 659 Bài viết
CMR: 2x^8+x^6-4x^4+5x^2-6x+ 2/x^2+1 luôn dương với mọi x thuộc R.
Lời Giải:
Ta Có:2x^8+x^6-4x^4+5x^2-6x+ 2/x^2+1 =2(x-1/x)^2+3(x-1)^2+x^6+2(x^4-1)^2 lớn hơn hoặc bằng 0
Dấu bằng xảy ra khi và chỉ khi đồng thời x=1 và x=0( vô lý).
Vậy ta có đpcm.

<span style="font-family: trebuchet ms" ,="" helvetica,="" sans-serif'="">Nỗ lực chưa đủ để thành công.


.if i sad, i do Inequality to become happy. when i happy, i do Inequality to keep happy.

#8
davildark

davildark

    Thượng sĩ

  • Thành viên
  • 223 Bài viết
Cho a,b,c là 3 cạnh tam giác chứng minh bất đẳng thức sau

$(a+b+c)(\frac{1}{2a+c}+\frac{1}{2b+a}+\frac{1}{2c+b})+1\geq 2(\frac{a+b}{2b+a}+\frac{b+c}{2c+b}+\frac{a+c}{2a+c})$


Bài giải



Nhân tung tóe và rút gọn ta được bất đẳng thức sau
$$\frac{3a+b+c}{2a+c}+\frac{3b+c+a}{2b+a}+\frac{3c+a+b}{2c+b}\geq 5$$
Bất đẳng thức trên đúng do
$$\frac{c}{2a+c}+\frac{a}{2b+a}+\frac{b}{2c+b}\geq \frac{(a+b+c)^2}{a^2+b^2+c^2+2ab+2ac+2bc}=1$$
$$\frac{3a+b}{2a+c}+\frac{3b+c}{2b+a}+\frac{3c+a}{2c+b}-3=\frac{a+b-c}{2a+c}+\frac{b+c-a}{2b+a}+\frac{a+c-b}{2c+b}\geq \frac{(\sum a+b-c)^2}{(a+b-c)(2a+c)}=\frac{(a+b+c)^2}{\sum a^2+2\sum ab}=1$$
Cộng các bất đẳng thức trên lại ta có đpcm
Dấu = xảy ra khi $a=b=c$

#9
WhjteShadow

WhjteShadow

    Thượng úy

  • Phó Quản lý Toán Ứng dụ
  • 1323 Bài viết
Toán thủ WhjteShadow xin ra đề (Chắc em nộp đề muộn nhất :P )
Đề bài: Ch0 các số thực dương $a,b,c,x,y,z$.Chứng minh bất đẳng thức:
$$\frac{bcx}{(x+y)(x+z)}+\frac{acy}{(x+y)(y+z)}+\frac{abz}{(x+z)(y+z)}\leq \frac{(a+b+c)^2}{4(x+y+z)}$$

Lời giải:
Đặt $m=x+y,n=y+z,k=z+x$ thì $m,n,k$ là độ dài 3 cạnh của 1 tam giác và bất đẳng thức đã ch0 tương đương với:
$$\frac{bc(m+k-n)}{mk}+\frac{ac(m+n-k)}{mn}+\frac{bc(n+k-m)}{nk}\leq \frac{(a+b+c)^2}{m+n+k}$$
$$\Leftrightarrow \frac{bc(m^2+k^2-n^2+2mk)}{mk}+\frac{ac(m^2+n^2-k^2+2mn)}{mn}+\frac{bc(n^2+k^2-m^2+2nk)}{nk}\leq (a+b+c)^2$$
$$\Leftrightarrow \frac{bc(m^2+k^2-n^2)}{mk}+\frac{ac(m^2+n^2-k^2)}{mn}+\frac{bc(n^2+k^2-m^2)}{nk}\leq a^2+b^2+c^2\,\,(*)$$
Nhưng mặt khác do $m,n,k$ là độ dài 3 cạnh tam giác nên (*) có thể viết lại thành:
$$2bc.cosN+2ac.cosK+2ab.cosM\leq a^2+b^2+c^2$$
Và đây là bất đẳng thức lượng giác quen thuộc nên ta có ĐPCM $\blacksquare$
“There is no way home, home is the way.” - Thich Nhat Hanh

#10
tran thanh binh dv class

tran thanh binh dv class

    Trung sĩ

  • Thành viên
  • 138 Bài viết
Đề : Chứng minh rằng với a,b,c>0 thì:
$\frac{1}{\sqrt{a^2+bc}}+\frac{1}{\sqrt{b^2+ca}}+\frac{1}{\sqrt{c^2+ab}}>\frac{2\sqrt{2}}{\sqrt{ab+bc+ca}}$

Lời giải:
Giả sử c=max{a,b,c}. Đặt $c=k^2(a+b)$ với $k>0$
Nếu $k\leq 1$ thì $c^2+ab\leq ab+bc+ca$. Kết hợp $b^2+ac\leq ab+bc+ca, a^2+bc\leq ab+bc+ca$ suy ra $f(a,b,c)\geq 3$
Xét khi k>1. Ta có: $c^2+ab\leq k^2(a+b)c+k^2ab\leq k^2(ab+bc+ca)$ (1)
$a^2+bc+b^2+ca\leq (a+b)^2+a(b+c)\leq (a+b)^2(1+k^2)=\frac{k^2+1}{k^2}(a+b)c<\frac{k^2+1}{k^2}(ab+bc+ca)$ (2)
Từ (1),(2) ta có $f(a,b,c)\geq \frac{1}{k}+\frac{2\sqrt{2}\sqrt{ab+bc+ca}}{\sqrt{a^2+bc+b^2+ca}}>\frac{1}{k}+\frac{2\sqrt{2}k}{k^2+1}=f(k)$
Ta chứng minh $f(k)>2\sqrt{2}$ với mọi k>0. Điều này có được vì:
$f(k)>2\sqrt{2}\Leftrightarrow \frac{1}{k}>\frac{2\sqrt{2}}{\sqrt{k^2+1}(\sqrt{k^2+1}+k)}\Leftrightarrow k^2+1+k\sqrt{k^2+1}>2\sqrt{2}k$
Bất đẳng thức cuối đúng do $k^2+1+k\sqrt{k^2+1}>2k^2+1>2\sqrt{2}k$

Hình đã gửi


#11
hoangtrunghieu22101997

hoangtrunghieu22101997

    Thượng sĩ

  • Thành viên
  • 206 Bài viết
Cho $a;b;c$ không âm .
Tìm $k$ lớn nhất sao cho bất đẳng thức sau đúng:
$\dfrac{a}{b+c}+\dfrac{b}{c+a}+\dfrac{c}{a+b} \ge \dfrac{3}{2}+\dfrac{k.max{(a-b)^2,(b-c)^2,(c-a)^2}}{ab+bc+ca}$

Hình đã gửi

Sự im lặng du dương hơn bất kỳ bản nhạc nào.


#12
tran thanh binh dv class

tran thanh binh dv class

    Trung sĩ

  • Thành viên
  • 138 Bài viết
Dạ thế để em post đáp án ạ :wacko: :wacko: :wacko: Em nhớ là gửi đề khác mà :ohmy: :ohmy:

Hình đã gửi


#13
Joker9999

Joker9999

    Thiếu úy

  • Thành viên
  • 659 Bài viết
Đề bài
$\sum a_{i}=n$ với $a_{i}\geq 0$ và $n\geq 4$.Cmr:
$\sum \frac{1}{n+1-a_{2}.a_{3}.....a_{n}}\leq 1$

<span style="font-family: trebuchet ms" ,="" helvetica,="" sans-serif'="">Nỗ lực chưa đủ để thành công.


.if i sad, i do Inequality to become happy. when i happy, i do Inequality to keep happy.

#14
huy thắng

huy thắng

    Hạ sĩ

  • Thành viên
  • 97 Bài viết
Cho 3 số thực a,b,c $>$ 0
chứng minh :
$6ab\left ( 7b-2c \right ) + 6bc\left ( 7c-2b \right ) + 6ac\left ( 7a-2c \right ) + 5\left ( a^{3} + b^{3} + c^{3} \right ) \geq 105abc$

Hình đã gửi


#15
nguyensidang

nguyensidang

    Binh nhất

  • Thành viên
  • 32 Bài viết
Cho a,b,c dương thỏa mãn $a^{3}+b^{3}+c^{3}=3$
Chứng minh rằng:
F=$\sum \frac{bc}{a\sqrt{a}}\geq 3$

#16
tranhuuquoc1999

tranhuuquoc1999

    Lính mới

  • Thành viên
  • 1 Bài viết

oh






1 người đang xem chủ đề

0 thành viên, 1 khách, 0 thành viên ẩn danh